rzaman
Thanks Received: 4
Vinny Gambini
Vinny Gambini
 
Posts: 10
Joined: October 23rd, 2010
 
 
 

Q21 - Newspaper editor: Law enforcement experts

by rzaman Wed Mar 30, 2011 5:15 pm

I would really appreciate if someone could please explain this problem to me and correct my work on it.

I diagrammed the problem as:

Premises:
Gambling laws are impossible to enforce
+
~Effective -> ~Law

Conclusion:
~Law (gambling)

Since the argument presents the premise that if a law is not effective, it should not be a law, the author has to assume that gambling laws are not effective to conclude that gambling laws should not be law, right? There is however one more linking point that the argument probably needs to make or else the problem would be too easy. The author needs to link that gambling laws are not effective to the other premise presented (gambling laws are impossible to enforce. So the author needs to assume that gambling laws are not effective because they are impossible to enforce, which is what answer choice A is basically saying.

Is my line of reasoning correct? Can someone explain why it is necessary to link gambling laws are ineffective to gambling laws are impossible to enforce. If there was an answer choice for this question that just said "Gambling laws fail to be effective" would that be a correct answer choice?
 
giladedelman
Thanks Received: 833
LSAT Geek
 
Posts: 619
Joined: April 04th, 2010
 
This post thanked 2 times.
 
 

Re: Q21 - Newspaper editor: Law enforcement experts

by giladedelman Sat Apr 02, 2011 6:31 pm

Very good post!

So, your reasoning is 99% accurate. Let's walk through this question one more time.

Like you said, we have two premises that lead to our conclusion, so the core looks like this:

anti-gambling laws unenforceable
if a law is ineffective, shouldn't be a law --> anti-gambling laws shouldn't exist

Now, again as you pointed out, the gap here is pretty clear. The only thing we know about anti-gambling laws is that they're unenforceable. The only way we know whether a law should exist is whether it's effective. So we can expect the assumption to connect "unenforceable" to "ineffective" -- something like, if it's unenforceable, it's not effective. (The only thing I didn't like in your explanation is where you said "the author needs to assume that gambling laws are not effective because they are impossible to enforce." We don't need a causal connection. But I'm nitpicking.)

(A) is correct because if no effective law is unenforceable, no unenforceable law is effective, so the anti-gambling laws are ineffective and we should get rid of them.

(B) is the negated version of what we want. It doesn't tell us anything useful about unenforceable laws.

(C) is just a restatement of the premise that anti-gambling laws are unenforceable.

(D) and (E) are both out of scope because whether citizens agree doesn't affect our core whatsoever.

Now, notice that this question asks, "Which one of the following, if assumed, allows the argument’s conclusion to be properly drawn?" That means we're looking for a sufficient assumption, one that gets us all the way to the conclusion.

So, to answer your question, yes: if answer (A) said "anti-gambling laws are ineffective," that would be correct, because it would be enough to get us to the conclusion. But you were also right to prepare to find an assumption that linked the two premises, since that's the tendency on assumption family questions.

Does that satisfy you? Good explanation.
 
rzaman
Thanks Received: 4
Vinny Gambini
Vinny Gambini
 
Posts: 10
Joined: October 23rd, 2010
 
 
 

Re: Q21 - Newspaper Editor: Law enforment experts, as well as mo

by rzaman Sun Apr 03, 2011 7:57 pm

Thanks for catching that unfortunate causal connection I made between the premises! Since this is a sufficient assumption question, I’m guessing that’s why the language of the correct answer is so broad. They could have said "No effective gambling law is unenforceable," which still connects the two premises but they broadened it to include all laws. Thanks for your help!
 
peg_city
Thanks Received: 3
Forum Guests
 
Posts: 152
Joined: January 31st, 2011
Location: Winnipeg
 
 
trophy
First Responder
 

Re: Q21 - Newspaper Editor: Law enforment experts, as well as mo

by peg_city Mon May 16, 2011 1:32 pm

Can one of the geeks explain to me why the inference we are supposed to draw is....


Gambling laws unenforceable -> gambling laws ineffective

instead of it being backwards

Gambling laws ineffective -> Gambling laws enforceable


I seem to be getting caught up on this. The conditional logic is over two sentences and how to combine the sentences into a chain is puzzling me.

Thanks Very Much
 
giladedelman
Thanks Received: 833
LSAT Geek
 
Posts: 619
Joined: April 04th, 2010
 
This post thanked 1 time.
 
 

Re: Q21 - Newspaper Editor: Law enforment experts, as well as mo

by giladedelman Fri May 20, 2011 12:50 pm

Ha, I didn't notice until now that I'm listed as an "LSAT Geek"! And here I always thought I was the cool LSAT instructor ...

Anyway, here's the deal with this one. We're told that gambling laws are unenforceable. Then we're told that we should scrap these laws, because ineffective laws should get scrapped.

But wait a minute! When did it ever say these laws were ineffective? All we know is that they're unenforceable! So the big gap here is between those two terms; the argument assumes that if a law is unenforceable, it's also ineffective. If we assume that, then we do have to conclude that we should scrap the gambling laws.

If we assume the reverse, that all ineffective laws are unenforceable, it doesn't help us because we already know the gambling laws are unenforceable; we need to go from there to ineffective.

Does that make sense? Notice that I didn't really need to bring formal logic into this one. I just broke it down by premises and conclusion and looked for the gap. I tend to reserve conditional logic for when there are a bunch of obvious conditional triggers.

Hope that helps!
 
peg_city
Thanks Received: 3
Forum Guests
 
Posts: 152
Joined: January 31st, 2011
Location: Winnipeg
 
 
trophy
First Responder
 

Re: Q21 - Newspaper Editor: Law enforment experts, as well as mo

by peg_city Sat Jul 16, 2011 3:43 pm

giladedelman Wrote:Ha, I didn't notice until now that I'm listed as an "LSAT Geek"! And here I always thought I was the cool LSAT instructor ...

Anyway, here's the deal with this one. We're told that gambling laws are unenforceable. Then we're told that we should scrap these laws, because ineffective laws should get scrapped.

But wait a minute! When did it ever say these laws were ineffective? All we know is that they're unenforceable! So the big gap here is between those two terms; the argument assumes that if a law is unenforceable, it's also ineffective. If we assume that, then we do have to conclude that we should scrap the gambling laws.

If we assume the reverse, that all ineffective laws are unenforceable, it doesn't help us because we already know the gambling laws are unenforceable; we need to go from there to ineffective.

Does that make sense? Notice that I didn't really need to bring formal logic into this one. I just broke it down by premises and conclusion and looked for the gap. I tend to reserve conditional logic for when there are a bunch of obvious conditional triggers.

Hope that helps!

I just want to make sure I have the conditional reasoning down.

GL = Gambling laws
L = Law

GL -> Unenforceable

Ineffective -> L~

Conclusion

GL -> L~ (??)

Therefore, Unenforceable -> Ineffective or Effective -> enforceable

Thanks again
 
giladedelman
Thanks Received: 833
LSAT Geek
 
Posts: 619
Joined: April 04th, 2010
 
This post thanked 1 time.
 
 

Re: Q21 - Newspaper Editor: Law enforment experts, as well as mo

by giladedelman Wed Jul 20, 2011 12:43 pm

Yep, looks good!
 
mxl392
Thanks Received: 0
Forum Guests
 
Posts: 22
Joined: July 16th, 2012
 
 
 

Re: Q21 - Newspaper editor: Law enforcement experts

by mxl392 Wed Jul 25, 2012 12:35 pm

It makes perfect sense to me why A is the right answer. I sat for a long time between A and C. I couldn't explain why C is wrong. It's true that C simply restates a claim in the stimulus. But isn't that claim essential to the argument? Wouldn't that make C an essential assumption?

You can't say that C isn't an assumption, could you? If C were false (using the negative rule), the argument would be invalid.
User avatar
 
ManhattanPrepLSAT1
Thanks Received: 1909
Atticus Finch
Atticus Finch
 
Posts: 2851
Joined: October 07th, 2009
 
 
 

Re: Q21 - Newspaper editor: Law enforcement experts

by ManhattanPrepLSAT1 Thu Jul 26, 2012 1:14 pm

By definition, assumptions are unstated. So answer choice (C) is not an assumption, but rather a premise. So no, you cannot say that answer choice (C) is an assumption.

Hope that helps!
 
jamiejames
Thanks Received: 3
Atticus Finch
Atticus Finch
 
Posts: 116
Joined: September 17th, 2011
 
 
 

Re: Q21 - Newspaper Editor: Law enforment experts, as well as mo

by jamiejames Sun Aug 25, 2013 3:53 pm

giladedelman Wrote:Ha, I didn't notice until now that I'm listed as an "LSAT Geek"! And here I always thought I was the cool LSAT instructor ...

Anyway, here's the deal with this one. We're told that gambling laws are unenforceable. Then we're told that we should scrap these laws, because ineffective laws should get scrapped.

But wait a minute! When did it ever say these laws were ineffective? All we know is that they're unenforceable! So the big gap here is between those two terms; the argument assumes that if a law is unenforceable, it's also ineffective. If we assume that, then we do have to conclude that we should scrap the gambling laws.

If we assume the reverse, that all ineffective laws are unenforceable, it doesn't help us because we already know the gambling laws are unenforceable; we need to go from there to ineffective.

Does that make sense? Notice that I didn't really need to bring formal logic into this one. I just broke it down by premises and conclusion and looked for the gap. I tend to reserve conditional logic for when there are a bunch of obvious conditional triggers.

Hope that helps!


quick point of clarification. When I prephrased my answer, it came out this way:

The gap here is between unenforceable laws, and the law being ineffective.

Answer:

This law is impossible to enforce.

When a law is ineffective, it shouldn't be a law.

No law that is unenforceable is effective

Therefore there shouldn’t be a law against it.

_________

my answer switched around unenforceable and effective, is that wrong?
User avatar
 
ManhattanPrepLSAT1
Thanks Received: 1909
Atticus Finch
Atticus Finch
 
Posts: 2851
Joined: October 07th, 2009
 
This post thanked 1 time.
 
 

Re: Q21 - Newspaper editor: Law enforcement experts

by ManhattanPrepLSAT1 Mon Aug 26, 2013 1:35 pm

Nice work jamiejames! Actually, you didn't switch around the assumption.

You phrased the assumption as, "if not enforceable, then not effective." They phrased the assumption as "No unenforceable law is effective."

Those two statements are noted the same way:

~Enforceable --> ~Effective


Hope that helps!
 
cwolfington
Thanks Received: 4
Jackie Chiles
Jackie Chiles
 
Posts: 29
Joined: May 15th, 2014
 
 
 

Re: Q21 - Newspaper editor: Law enforcement experts

by cwolfington Thu Jun 12, 2014 12:27 am

The final premise is diagrammed: ~effective --> ~law; and since the conclusion states that prohibition should not be legal, we know that prohibition must be ~effective, because that would make the conclusion valid.

And what do we know about prohibition so far? We know that it's ~enforceable. So, the author's assumption must be: ~enforceable--> ~effective.

The full diagram, assumption included, is: ~enforceable --> ~effective --> ~law

Notice that this is a sufficient assumption question, and that the assumption takes the role of the sufficient condition.

Answer A) states the contrapositive of the assumption, which is: effective --> enforceable
 
marokh9
Thanks Received: 0
Forum Guests
 
Posts: 7
Joined: October 22nd, 2013
 
 
 

Re: Q21 - Newspaper editor: Law enforcement experts

by marokh9 Mon Aug 18, 2014 4:15 pm

Hello,

I have read all of the above comments and I just cannot wrap my head around A and B. I originally picked B and I still do not know why its the wrong answer choice, or why A is the better answer.

So, I know that:

Gambling --> impossible to enforce
Law ~effective --> ~law

Therefore: Gambling --> ~law
-----------------------------------

so at this point I know that: Gambling --> ~enforceable --> ~effective --> ~law

I think the problem I am having is understanding what (A) and (B) are actually saying... >_<

(A) effective --> enforceable ??
(B) enforceable --> effective ??


ohhhhh okay I think I get it now after typing it out on here.. (B) is a wrong contrapositive..can someone help clarify my understanding please? Is my diagram correct?

Thanks in advance!
User avatar
 
WaltGrace1983
Thanks Received: 208
Atticus Finch
Atticus Finch
 
Posts: 837
Joined: March 30th, 2013
 
 
trophy
Most Thanked
trophy
Most Thankful
trophy
First Responder
 

Re: Q21 - Newspaper editor: Law enforcement experts

by WaltGrace1983 Thu Sep 18, 2014 9:45 am

marokh9 Wrote:Hello,

I have read all of the above comments and I just cannot wrap my head around A and B. I originally picked B and I still do not know why its the wrong answer choice, or why A is the better answer.

So, I know that:

Gambling --> impossible to enforce
Law ~effective --> ~law

Therefore: Gambling --> ~law
-----------------------------------

so at this point I know that: Gambling --> ~enforceable --> ~effective --> ~law

I am probably misunderstanding what you are saying here but you actually do not know this. You are making it seem like we have a connection that we do not. We do not have anything connecting ~enforceable → ~effective. THAT is the problem; THAT is the assumption we need.

I think the problem I am having is understanding what (A) and (B) are actually saying... >_<

(A) effective --> enforceable ??
(B) enforceable --> effective ??


Yes! You got it right!

(A) "No effective law is unenforceable."
    - IF it's effective then it CANNOT BE unenforceable. So (effective → ~unenforceable).
    - In other words, (effective → enforceable).
    - We can take the contrapositive here and get (~enforceable → ~effective).
    - We know that the laws are ~(enforceable)
    - Therefore, answer choice (A) would tell us that they are ~(effective)
    - Perfect!


(B) "All enforceable laws are effective"
    - Uh oh! We only know that these laws are ~(enforceable). So what can we do with a statement talking about (enforceable) laws? The answer is nothing. (B) is irrelevant.


Do you get it now?
User avatar
 
WaltGrace1983
Thanks Received: 208
Atticus Finch
Atticus Finch
 
Posts: 837
Joined: March 30th, 2013
 
 
trophy
Most Thanked
trophy
Most Thankful
trophy
First Responder
 

Re: Q21 - Newspaper editor: Law enforcement experts

by WaltGrace1983 Thu Sep 18, 2014 9:47 am

A few questions for the geeks here.

(1) Would a correct necessary assumption be, "If some gambling laws are not enforceable, they are not effective?"

(2) Could (C) be correct if we said "No legal prohibitions against gambling are effective." Now I know this would be a bit different from the typical connect-the-premises answer choice, but it seems like it would still work.
 
christine.defenbaugh
Thanks Received: 585
Atticus Finch
Atticus Finch
 
Posts: 536
Joined: May 17th, 2013
 
This post thanked 1 time.
 
 

Re: Q21 - Newspaper editor: Law enforcement experts

by christine.defenbaugh Tue Sep 23, 2014 3:27 am

Welcome back, WaltGrace1983! 8-)

1) Yes, that would be a valid Necessary Assumption, though I would expect it to be worded a bit differently. Perhaps "For at least some gambling laws, if they are not enforceable, they are ineffective."

2) Yes, this would be a perfectly valid Sufficient Assumption. It's just too easy. :p

Great explanation above, btw. I find that the No X are Y construction is tricky for a lot of people to diagram at first.

marokh9, it helps to remember that the "no" is not attached to either element, but rather is saying that these two elements cannot be together.

"No roses are purple" means that 'rose' and 'purple' can never co-exist. So if I have one, I can't have the other. That means I can translate it quickly as either:
    If rose, then not purple OR
    If purple, then not rose

And since those two notations are contrapositives of each other, I'm good to go!

I hope this helps clarify a few things for both of you!
 
LauraS737
Thanks Received: 0
Vinny Gambini
Vinny Gambini
 
Posts: 19
Joined: May 14th, 2017
 
 
 

Re: Q21 - Newspaper editor: Law enforcement experts

by LauraS737 Sun Jun 04, 2017 9:51 pm

I'm having a hard time diagramming this. Can someone help?
Here is how I diagrammed it initially:

1) No Enforceable
2) No Effective --> No Law
------------------------------------
Conclusion: No Law (Gambling)

Link the premise to the conclusion:

No Effective --> No Law (Gambling).

This is where my confusion is :? Should No Enforceable go before No Effective, or after? To be clearer should it be:

No effective --> No Enforceable --> No Law (Gambling)

OR should it be:

No Enforceable --> No Effective --> No Law (Gambling)

Can someone explain how the ordering when linking the premise with the conclusion to find the assumption?
 
ghorizon09
Thanks Received: 0
Vinny Gambini
Vinny Gambini
 
Posts: 14
Joined: November 30th, 2016
 
 
 

Re: Q21 - Newspaper editor: Law enforcement experts

by ghorizon09 Wed Jul 05, 2017 4:26 pm

A. No effective law is unenforceable - This statement must be translated to All effective laws are enforceable. Then reverse and negate to get the contrapositive.

- Enforceable ----> - Effective

We can feed this into our conditional statement in the passage. When - Effective ----> - Law

we combine the two statements

By combining, we see that if a law is not enforceable then it's not effective, we know from the passage if a law is not effective it should not be a law.

Not Enforceable ----> Not Effective ------> Not Law
 
VendelaG465
Thanks Received: 0
Elle Woods
Elle Woods
 
Posts: 66
Joined: August 22nd, 2017
 
 
 

Re: Q21 - Newspaper editor: Law enforcement experts

by VendelaG465 Mon Oct 23, 2017 4:14 pm

Is choice A basically the contrapositive of the statement "when a law fails to be effective, it should not be law." ---> it should be law if it is effective?
User avatar
 
ohthatpatrick
Thanks Received: 3808
Atticus Finch
Atticus Finch
 
Posts: 4661
Joined: April 01st, 2011
 
 
 

Re: Q21 - Newspaper editor: Law enforcement experts

by ohthatpatrick Wed Oct 25, 2017 1:46 pm

Consider any claim of the form "No A's are B".

No lions can speak English.


Shall we represent that as
If you're not a lion, then you can speak English
or
If you are a lion, then you can't speak English

The 2nd one is what we want. "No A's are B" = If A, then not-B

So translate (A) accordingly:
If you're an effective law, then you're not-unenforceable.
i.e. If you're an effective law, then you're enforceable.

The contrapositive of that rule would be,
If you're not-enforceable, then you're not-effective law.

According to our evidence, "when a law fails to be effective it shouldn't be a law."
If we add the rule from (A), we know that
If a law is not enforceable, it's not effective, and thus it shouldn't be a law.

We were told that gambling laws are not enforceable, so this chained together rule lets us derive that gambling laws shouldn't be laws.